LSAT and Law School Admissions Forum

Get expert LSAT preparation and law school admissions advice from PowerScore Test Preparation.

 Administrator
PowerScore Staff
  • PowerScore Staff
  • Posts: 8916
  • Joined: Feb 02, 2011
|
#23066
Complete Question Explanation

Assumption. The correct answer choice is (D)

The author recommends that governments subsidize high-quality day care because subsidies are the only way to ensure such day care is available to everyone, and governments have an obligation to improve the well-being of all children in society. Do you notice a discrepancy between the principle advanced in the first sentence of the stimulus and its conclusion? What if the wide-spread availability of day care only benefited adults? Because the government's obligation towards children would not be met in that case, financing day care may be unnecessary. In other words, the argument assumes that making day care available to everyone will benefit at least some children. This, essentially, is answer choice (D).

Answer choice (A): At first glance, this is an attractive answer choice. If the only governments that take an interest in the well-being of children are those that subsidize high-quality day care, and each government must do all that it can to improve the well-being of the children in the society it governs, then subsidizing day care would be a logical conclusion. This, however, is not a Justify the Conclusion question! Our job in an Assumption question is not to prove the conclusion, but rather to find an answer choice that would disprove the conclusion if removed from it. What if it were possible to take an interest in the well-being of children without subsidizing high-quality day care? This would not necessarily weaken the argument, because subsidizing day care could still be a valuable practice. This answer choice is incorrect.

Answer choice (B): Even if government subsidy of day care led to the elimination of benefits for adults, the author is only concerned with the well-being of children. While this answer choice would lend additional support to subsidizing high-quality day care, it is not essential to the conclusion. This answer choice is incorrect.

Answer choice (C): It is a given that day care would only be available to everyone if it's subsidized. Restricting the coverage of such a subsidy is not essential to the conclusion. This answer choice is incorrect.

Answer choice (D): This is the correct answer choice. It is necessary that at least some children benefit from day care. If none of them do, the government initiative would not satisfy the goal of improving the well-being of children.

Answer choice (E): The relative efficiency of government-provided services is irrelevant to this argument. This answer choice is incorrect.
 180nce
  • Posts: 12
  • Joined: May 18, 2015
|
#19121
I am a bit confused as to how answer choice A is wrong. I see why answer choice D is correct because it addresses the gap that day care will actually benefit the children, but I can't seem to eliminate answer choice A.


Any help would be appreciated.

Thanks!
 Robert Carroll
PowerScore Staff
  • PowerScore Staff
  • Posts: 1787
  • Joined: Dec 06, 2013
|
#19124
180,

Since there is a gap here, you are right to look for answer choices that close the gap. Answer choice (A) does not close that gap, and it is not an assumption that is necessary for the argument. "Taking an interest in well-being" and "improving well-being" are not the same thing - an entity could take an interest in something but be totally ineffective, in which case the gap here would not be closed. The professor's argument depends on the solution offered actually having an effect on the problem. For the government to take an interest does not yet rise to the level of an effective solution.

Robert Carroll
 sgd2114
  • Posts: 23
  • Joined: Jul 14, 2017
|
#37290
Hi,

I understand why Answer Choice (D) is correct and why Answer Choice (A) is incorrect. However, in the original explanation above, Administrator hinted that Answer Choice (A) would be a correct answer for a Justify question. Any help in walking through how exactly (A) makes the conclusion valid would be greatly appreciated.

Thank you.
 Luke Haqq
PowerScore Staff
  • PowerScore Staff
  • Posts: 747
  • Joined: Apr 26, 2012
|
#37299
Hi sgd2114!

If the only governments that take an interest in the well-being of children are those that subsidize high-quality day care (as (A) supposes), this is saying there is effectively only one way a government can "improve the well-being of all the children in the society it governs." And the first sentence is saying that governments should "improve the well-being of all the children in the society it governs." Thus (A) would make the conclusion--that governments should adopt high-quality day care--follow from the premises--i.e., it could work if this were a justify rather than an assumption question. Effectively, it's saying "X should do Y, and the only way to do Y is to first do Z. Therefore, X should first do Z."

Hope that helps!
 sgd2114
  • Posts: 23
  • Joined: Jul 14, 2017
|
#37360
Very helpful, thank you!
 lolaSur
  • Posts: 46
  • Joined: Nov 11, 2019
|
#72408
Hi!

I would like to make sure that I am understanding correctly why Answer A would work if the question was Justify the Conclusion. Additionally, I would like to confirm that I am understanding Answer choice A correctly.

Answer choice A states "Only governments that subsidize high quality day care take an interest in the well-being of all the children in the societies they govern."

I think Answer choice A is conditional, but I am having a hard time identifying what is the necessary part and what is the conditional part. Does the word "Only" here indicate necessity or does it indicate sufficiency as in "The Only governments" Can I diagram Answer A like this:

Gov't subsidizes day care :arrow: Gov't interest in children's well-being

At first glance my intuition told me that "only governments that subsidize" is the sufficient, but I am not sure because the word "only" is next to governments that subsidize.

Am I right to say that according to the logic of Answer A, if I know that a government subsidizes day care then it must be the case that the government cares about the well being of all children?


for my reference (L5 A/J/S q49)
 James Finch
PowerScore Staff
  • PowerScore Staff
  • Posts: 943
  • Joined: Sep 06, 2017
|
#72427
Hi Lola,

No, as a straight "only" this is a necessary condition indicator. We can abstract (A) out to make it make more sense: Only X do Y. Would this mean that if you know X then Y must be true? Not necessarily, as there could be X that don't do Y. Instead, what we know is that anything that does Y also is X. So Y :arrow: X, and thus:

Take an interest in the well-being of all the children in the societies they govern :arrow: Subsidize high-quality day care

So (A) would justify the conclusion, by starting off with the idea that governments taking an interest in children, therefore they should also subsidize day care.

Hope this clears things up!
User avatar
 aghartism
  • Posts: 21
  • Joined: Jul 11, 2023
|
#102452
Hi,

In the explanation above one finds:
Our job in an Assumption question is not to prove the conclusion, but rather to find an answer choice that would disprove the conclusion if removed from it.
Regarding this item, (D) does not disprove the conclusion if negated. The negation of (D) is that no children would benefit from high-quality day care. First, this is consistent with day care having a neutral effect on all children. Second, the argument in the stimulus does not assume that every government should only do things that improve the well-being of children, just that every government do all that it can.

I'm trying to develop an air-tight understanding of what the test-takers mean for an argument to "depend" on a certain assumption. This item shows that it cannot mean what I thought it meant. Any clarification would be appreciated!

Get the most out of your LSAT Prep Plus subscription.

Analyze and track your performance with our Testing and Analytics Package.